LSAT and Law School Admissions Forum

Get expert LSAT preparation and law school admissions advice from PowerScore Test Preparation.

 Administrator
PowerScore Staff
  • PowerScore Staff
  • Posts: 8916
  • Joined: Feb 02, 2011
|
#26463
Complete Question Explanation

Question #6: Justify, SN. The correct answer choice is (B).

In this argument, the author presents several conditional statements and draws a conclusion that requires further justification. Since the conclusion appears in the middle of the stimulus, it might be helpful to consider the author’s argument slightly reordered:
Premise: Board :arrow: Undergraduate degree
Premise: Felony :arrow: Board (contrapositive: Board :arrow: Felony)
Premise: Felony Murray

Conclusion: Admin Murray
The question stem asks us to identify a statement that, if assumed, would allow the conclusion to be properly drawn. Despite the word “assumed” in the stem, this is a Justify question because our job is not to identify a statement upon which the argument depends, but rather to prove the conclusion by adding a piece of information to the premises. The sufficient condition indicator (“if”) in the question stem is a reminder that you must select an answer that is sufficient to prove the conclusion by using the Justify Formula:
  • Premises + Answer choice :arrow: Conclusion

The first thing you should notice is that the conclusion introduces a new element into this argument: the position of Executive Administrator. At the very least, the correct answer choice must “tie” that element to the rest of the argument, which helps eliminate answer choice (A).

Second, consider the sort of the conclusion that the premises would justify. According to the premises, appointment to the board has two requirements: you must have an undergrad degree, and you cannot be a felon. Given that Murray is a felon, he clearly cannot be appointed to the board. From this fact, the author goes a step further to conclude that Murray cannot be accepted for the position of Exec. Administrator. To justify the conclusion, we need to establish that if someone cannot be appointed to the board, then that person cannot be an administrator:
  • Justify: Board :arrow: Admin
Or, by the contrapositive:
  • Justify: Admin :arrow: Board
This prephrase agrees with answer choice (B).

Answer choice (A): This answer choice does not mention the qualifications for being accepted for the position of Executive Administrator. This is sufficient to eliminate it from consideration.

Answer choice (B): This is the correct answer choice, as it is the contrapositive of our prephrase. When added to the premises, this answer choice justifies the conclusion:
  • Felony Murray :arrow: Board Murray :arrow: Admin Murray
Answer choice (C): Just because an undergrad degree is not necessary for the Exec admin position does not mean that someone who has a degree cannot be accepted to that position.

Answer choice (D): A conclusion about someone’s rejection cannot be proved with a hypothetical suggesting the conditions under which he would have been accepted.

Answer choice (E): Even if we assume that the felony charge is relevant to the duties of an Exec admin, this answer choice merely states an assumption upon which the argument depends, not a premise sufficient to prove the conclusion.
 Katya W
  • Posts: 42
  • Joined: Dec 03, 2019
|
#72410
Hi, thank you for the explanation of this question. However, I’m looking for some further clarification on why D is not also a good answer. I didn't find your explanation for D that "...[a] conclusion about someone’s rejection cannot be proved with a hypothetical suggesting the conditions under which he would have been accepted" was entirely valid because if we draw out the formal logic equation for D we get: /FC(no felony conviction)-->EA(Executive administrator). Which CAN be concluded using the formal logic equation we created from the stimulus about the conclusion, which was FC-->/EA. Now, the contrapositive to that equation is /FC-->EA, which is exactly the equation in D. So given this, I do not understand how D cannot be a legitimate answer to this question. We do have the information presented in D based on what the author told us, and even based on our own formal logic equations. Yes, I understand the notion of "we can’t assume that there are no other factors that may bar Murray from EA", however, we are constantly being taught to use the information provided to us in the stimulus, and not try to come up with our own presuppositions.

Please help me understand! This has been tripping me up for hours.

Thank you, Katya
 James Finch
PowerScore Staff
  • PowerScore Staff
  • Posts: 943
  • Joined: Sep 06, 2017
|
#72423
Hi Katya,

This is a Justify question, which means the correct answer will add in a premise that, when combined with the other premises, make the conclusion 100% indisputably true. These questions can be solved mechanically, as they always contain a new element in the conclusion that is not present in any of the premises, and this allows them to always be accurately Prephrased by tying that new element in the conclusion to a relevant premise. The trick is to identify the relevant premise that acts as a sort of dangling thread or loose end and tie that to the conclusion, and this is almost always done with conditional logic chains.

Here, the new element in the conclusion is the Executive Administrator position, which isn't mentioned in the premises; don't confuse it with the similarly named executive board. So that's what need to get to, and will serve as our necessary condition:

Executive Administrator (EA)

which in the context of the conclusion will be negated:

EAMurray

So what are the relevant premises? We're given a conditional statement:

Felony (F) :arrow: Executive Board (EB)

and then told that Murray has a felony. So what we know so far from the premises is:

FM :arrow: EBM

Which means what we need to do is tie it all together to make a chain of:

FM :arrow: EBM :arrow: EAM

How do we do that? We fill in the missing assumption:

EB :arrow: EA

or its contrapositive:

EA :arrow: EB

(B) is exactly that contrapositive, and thus correct.

(D) says:

F :arrow: EA

which is a Mistaken Negation of the argument in the stimulus that we're trying to prove.

Hope this clears things up!

Get the most out of your LSAT Prep Plus subscription.

Analyze and track your performance with our Testing and Analytics Package.